Exploring Zee's Quantum Field Theory: Unraveling p. 31

In summary, the author provides a quick overview of quantum field theory, but the explanation is not very clear. There is a discussion of the action and its Fourier transform, but no explanation of how the Green's function is related to the propagator or how the delta and lambda come into the equation. Further questions about the origin of the Kronecker delta and why the indices \lambda and \mu are used in the Green's function equation are not answered. It seems that the equation might be justified by some theorem, but the author is not able to provide a reference for this.
  • #1
jdstokes
523
1

Homework Statement



A. Zee Quantum Field theory in a nutshell, p. 31. There is painfully little explanation on this page.

I'm okay with the action:

[itex]S(A) = \int d^4 x \mathcal{L} = \int d^4 x\{ \frac{1}{2}A_\mu [(\partial^2 +m^2)g^{\mu \nu}-\partial^\mu\partial^\nu]A_\nu + A_\mu J^\mu \}[/itex]

where I'm assuming [itex]\partial^2 = \partial^\mu\partial_\mu[/itex], but how does he jump from that to the Green's function

[itex][(\partial^2+m^2)g^{\mu\nu}-\partial^\mu\partial^\nu]D_{\nu\lambda}(x) = \delta^\mu_\lambda \delta^{(4)}(x)[/itex]?

Where did the Kronecker delta and the subscript \lambda come from??
 
Physics news on Phys.org
  • #2
It happens that the propagator is the inverse of the operator appearing in the quadratic term in the Lagrangian. I haven't seen a proof, but i believe there exists one. Thus, what you simply have in your equation is

Quadratic term * Propagator = 1

the delta and the lambda are there to satisfy this assumption.
 
  • #3
Thanks for your reply.

I understand where the equation comes from but I don't understand the presence of the Kronecker delta [itex]\delta^\mu_\lambda[/itex] nor why the green's function is indexed by [itex]\nu,\lambda[/itex].
 
Last edited:
  • #4
If we put [itex]\lambda = \mu[/itex] we get

[itex][(\partial^2+m^2)g^{\mu\nu}-\partial^\mu\partial^\nu]D_{\nu\mu}(x) = \delta^{(4)}(x)[/itex]

which makes a little more sense since we're now summing over all indices. Is there any reason for summing over the [itex]\nu[/itex] index as opposed to [itex]\mu[/itex] when [itex]\lambda\neq\mu[/itex].

Further question: Fourier transforming gives

[itex][-(k^2-m^2)g^{\mu\nu}+k^\nu k^\mu]D_{\nu\lambda}(k) = \delta^\mu_\lambda[/itex].

I don't see how to get from this to Eq. (3):

[itex]D_{\nu \lambda}(k)= -\frac{g_{\nu\lambda} + k_\nu k_\lambda /m^2}{k^2-m^2}[/itex].

Am I missing something obvious?
 
Last edited:
  • #5
jdstokes said:
If we put [itex]\lambda = \mu[/itex] we get

[itex][(\partial^2+m^2)g^{\mu\nu}-\partial^\mu\partial^\nu]D_{\nu\mu}(x) = \delta^{(4)}(x)[/itex]

which makes a little more sense since we're now summing over all indices. Is there any reason for summing over the [itex]\nu[/itex] index as opposed to [itex]\mu[/itex] when [itex]\lambda\neq\mu[/itex].

Further question: Fourier transforming gives

[itex][-(k^2-m^2)g^{\mu\nu}+k^\nu k^\mu]D_{\nu\lambda}(k) = \delta^\mu_\lambda[/itex].

I don't see how to get from this to Eq. (3):

[itex]D_{\nu \lambda}(k)= -\frac{g_{\nu\lambda} + k_\nu k_\lambda /m^2}{k^2-m^2}[/itex].

Am I missing something obvious?


To find [itex]D_{\nu \lambda}[/itex] from the equation, write the most general possible form, which is [tex] A(k) k_{\nu} k_{\lambda} + B(k) g_{\nu \lambda} [/tex] and solve for A and B.
 
  • #6
*Head explodes*. Is there some way you can justify that [tex]D_{\nu\lambda} = A(k) k_{\nu} k_{\lambda} + B(k) g_{\nu \lambda} [/itex]??

I wish I knew more about Green's functions...

Is it just because there's some theorem which says that the Green's function must be of the same general form as the operator?
 
Last edited:
  • #7
jdstokes said:
*Head explodes*. Is there some way you can justify that [tex]D_{\nu\lambda} = A(k) k_{\nu} k_{\lambda} + B(k) g_{\nu \lambda} [/itex]??

I wish I knew more about Green's functions...

Is it just because there's some theorem which says that the Green's function must be of the same general form as the operator?

It's simply that [tex] D_{\nu \lambda} [/tex] must be a tensor with two indices (a second rank tensor). the only things you have at your disposal to build D from are k and the metric. So that's the the most general thing you can write down, that's all.
 
  • #8
Hey nrqed,

Thanks. That makes things much clearer. I'm still troubled about how we're supposed to find A(k) and B(k), we have

[itex][-(k^2-m^2)g^{\mu\nu}+k^\nu k^\mu](A(k) k_{\nu} k_{\lambda} + B(k) g_{\nu \lambda}) = \delta^\mu_\lambda[/itex]

But now what? Expanding things out doesn't help much. It seems like we need some conditions on the behaviour of A and B for various k, but we aren't told anything? Where is the extra equation?
 
  • #9
Let [itex]D_{\nu\lambda}(k) = A(k) g_{\nu\lambda} + B(k) k_\nu k_\lambda[/itex].

[itex][-(k^2-m^2)g^{\mu\nu}+k^\nu k^\mu](A(k) g_{\nu\lambda} + B(k) k_\nu k_\lambda) = \delta^\mu_\lambda[/itex]
[itex][-(k^2-m^2)g^{\mu\nu}+k^\nu k^\mu](A(k) g_{\nu\lambda} + B(k) k_\nu k_\lambda) = 1[/itex]
[itex]-(k^2-m^2)g^{\mu\nu}g_{\nu\mu}A(k) + k^\mu k^\nu g_{\nu\mu}A(k) -(k^2 -m^2)g^{\mu\nu}k_\nu k_\mu B(k) + k^\mu k^\nu k_\nu k_\mu B(k)=1[/itex]
[itex]-(k^2-m^2)A(k) + k^2A(k) -(k^2 -m^2)k^2 B(k) + k^4 B(k)=1 \implies[/itex]
[itex]A(k) = \frac{1}{m^2}- k^2 B(k)\implies[/itex]

[itex]D_{\nu\lambda} = \frac{g_{\nu\lambda}}{m^2} - g_{\nu\lambda}k^\nu k_\nu B(k) + B(k) k_\nu k_\lambda = \frac{g_{\nu\lambda}}{m^2}[/itex]

What am I missing here?
 
Last edited:
  • #10
You are multiplying 2 matrices together, the first with indices [itex]\mu\nu[/itex] and the second with indices [itex]\nu\lambda[/itex], and you are summing over [itex]\nu[/itex]. You should not set [itex]\mu=\lambda[/itex], because you lose information when you do this.

Then, in your last line, you illegally use the index nu twice in the second term; [tex]g_{\nu\lambda}k^\nu k_\nu[/tex] should be [tex]g_{\nu\lambda}k^\rho k_\rho[/tex].
 
Last edited:
  • #11
Second attempt:

Assume [itex]A(k) = -\frac{1}{k^2-m^2}[/itex]. Then

[itex][-(k^2-m^2)g^{\mu\nu}+k^\nu k^\mu](A(k) g_{\nu\lambda} + B(k) k_\nu k_\lambda) = \delta^\mu_\nu - \frac{k^\mu k_\lambda}{k^2-m^2} + Bk_\nu k_\lambda[-(k^2-m^2)g^{\mu\nu}+ k^\mu k^\nu][/itex]

[itex][-(k^2-m^2)g^{\mu\nu}+k^\nu k^\mu](A(k) g_{\nu\lambda} + B(k) k_\nu k_\lambda) = \delta^\mu_\nu - \frac{k^\mu k_\lambda}{k^2-m^2} + B[-(k^2-m^2)k^\mu k_\lambda+ k^2k^\mu k_\lambda][/itex]

[itex][-(k^2-m^2)g^{\mu\nu}+k^\nu k^\mu](A(k) g_{\nu\lambda} + B(k) k_\nu k_\lambda) = \delta^\mu_\nu - \frac{k^\mu k_\lambda}{k^2-m^2} + B[k^\mu k_\lambda m^2][/itex]

Therefore [itex]B = \frac{1/m^2}{k^2-m^2}[/itex].

But I hate the fact that I needed to assume the form of A(k). Is there are more elegant way to derive this?
 
  • #12
You don't have to assume it.

[tex][-(k^2-m^2)g^{\mu\nu}+k^\mu k^\nu](A g_{\nu\lambda} + B k_\nu k_\lambda)=-(k^2-m^2)(A \delta^\mu{}_\lambda +Bk^\mu k_\lambda)+k^\mu(Ak_\lambda+Bk^2k_\lambda)=-(k^2-m^2)A \delta^\mu{}_\lambda +(A+Bm^2)k^\mu k_\lambda[/tex]

This is supposed to equal [tex]\delta^\mu{}_\lambda[/tex], so we must have [tex]-(k^2-m^2)A =1 [/tex] and [tex]A+Bm^2=0.[/tex]
 
  • #13
jdstokes said:
Let [itex]D_{\nu\lambda}(k) = A(k) g_{\nu\lambda} + B(k) k_\nu k_\lambda[/itex].

[itex][-(k^2-m^2)g^{\mu\nu}+k^\nu k^\mu](A(k) g_{\nu\lambda} + B(k) k_\nu k_\lambda) = \delta^\mu_\lambda[/itex]
[itex][-(k^2-m^2)g^{\mu\nu}+k^\nu k^\mu](A(k) g_{\nu\lambda} + B(k) k_\nu k_\lambda) = 1[/itex]
There was no reason to replace the [itex] \delta^\mu_\lambda [/itex] by one on the rhs there!
[itex]-(k^2-m^2)g^{\mu\nu}g_{\nu\mu}A(k) + k^\mu k^\nu g_{\nu\mu}A(k) -(k^2 -m^2)g^{\mu\nu}k_\nu k_\mu B(k) + k^\mu k^\nu k_\nu k_\mu B(k)=1[/itex]
And then you set lambda = mu but you should not have done that.

See Avodyne's post for the full derivation.


As an aside, it is instructive to do the same calculation for a massless A_\mu. Then you find out that there is no solution A(k) and B(k) satisfying the equation! This is because of gauge invariance; one must add a gauge-fixing term to get a solution. Here, it's not a problem because the massive field case does not have that gauge invariance.
 
  • #14
jdstokes said:
If we put [itex]\lambda = \mu[/itex] we get

IF you put [itex] \lambda=\mu [/itex] and sum, then [itex] \delta_{\mu}^{\mu} = 4 [/itex]
 

Related to Exploring Zee's Quantum Field Theory: Unraveling p. 31

What is Zee's Quantum Field Theory?

Zee's Quantum Field Theory is a theoretical framework used to describe the behavior of particles and their interactions in the quantum world. It combines quantum mechanics and special relativity to explain the fundamental forces of nature.

What is the significance of p. 31 in Zee's Quantum Field Theory?

Page 31 of Zee's Quantum Field Theory is significant because it marks the beginning of the chapter on gauge theories, which are essential in understanding the standard model of particle physics. This chapter introduces the concept of gauge invariance and its role in the theory.

How does Zee's Quantum Field Theory relate to other theories in physics?

Zee's Quantum Field Theory is a fundamental theory that builds upon and extends other theories in physics, such as quantum mechanics, special relativity, and gauge theories. It provides a more comprehensive framework for understanding the behavior of particles and their interactions.

What are some practical applications of Zee's Quantum Field Theory?

Zee's Quantum Field Theory has various practical applications in fields such as particle physics, condensed matter physics, and cosmology. It has been used to predict and explain phenomena such as particle decays, phase transitions, and the behavior of the early universe.

What are some common misconceptions about Zee's Quantum Field Theory?

One common misconception about Zee's Quantum Field Theory is that it is a complete theory of everything, which is not true. It is still a work in progress and has limitations, such as not being able to incorporate gravity. Another misconception is that it is too complex for non-scientists to understand, but there are many resources available to help explain the concepts in simpler terms.

Similar threads

  • Advanced Physics Homework Help
Replies
1
Views
387
Replies
5
Views
2K
  • Advanced Physics Homework Help
Replies
5
Views
3K
  • Advanced Physics Homework Help
Replies
10
Views
1K
  • Advanced Physics Homework Help
Replies
1
Views
731
  • Advanced Physics Homework Help
Replies
1
Views
871
Replies
1
Views
881
  • Advanced Physics Homework Help
Replies
1
Views
1K
  • Advanced Physics Homework Help
Replies
1
Views
1K
  • Advanced Physics Homework Help
Replies
9
Views
2K
Back
Top